Induction proof inequality

The name of the pictureThe name of the pictureThe name of the pictureClash Royale CLAN TAG#URR8PPP











up vote
0
down vote

favorite












So I got this induction proof question but I can't seem to make a logical statement in one part of it:



The question is , $a_n + 1 = 5 - frac6a_n + 2$ with
$a_1 = 1$ . Prove by induction that $a_n < 4$ for $n geq 1$



I reached up to the proof where I need to prove $a_k+1 <4$



Proof



$a_k <4 implies a_k + 2<6 $



The next step I want to put is:



$frac6 a_k +2 >1$



However I can only justify this statement if $a_k > -2$ but I can't seem to prove that or find any info in the question to suggest that it.



Can anyone help me with the proof or my theory?







share|cite|improve this question

























    up vote
    0
    down vote

    favorite












    So I got this induction proof question but I can't seem to make a logical statement in one part of it:



    The question is , $a_n + 1 = 5 - frac6a_n + 2$ with
    $a_1 = 1$ . Prove by induction that $a_n < 4$ for $n geq 1$



    I reached up to the proof where I need to prove $a_k+1 <4$



    Proof



    $a_k <4 implies a_k + 2<6 $



    The next step I want to put is:



    $frac6 a_k +2 >1$



    However I can only justify this statement if $a_k > -2$ but I can't seem to prove that or find any info in the question to suggest that it.



    Can anyone help me with the proof or my theory?







    share|cite|improve this question























      up vote
      0
      down vote

      favorite









      up vote
      0
      down vote

      favorite











      So I got this induction proof question but I can't seem to make a logical statement in one part of it:



      The question is , $a_n + 1 = 5 - frac6a_n + 2$ with
      $a_1 = 1$ . Prove by induction that $a_n < 4$ for $n geq 1$



      I reached up to the proof where I need to prove $a_k+1 <4$



      Proof



      $a_k <4 implies a_k + 2<6 $



      The next step I want to put is:



      $frac6 a_k +2 >1$



      However I can only justify this statement if $a_k > -2$ but I can't seem to prove that or find any info in the question to suggest that it.



      Can anyone help me with the proof or my theory?







      share|cite|improve this question













      So I got this induction proof question but I can't seem to make a logical statement in one part of it:



      The question is , $a_n + 1 = 5 - frac6a_n + 2$ with
      $a_1 = 1$ . Prove by induction that $a_n < 4$ for $n geq 1$



      I reached up to the proof where I need to prove $a_k+1 <4$



      Proof



      $a_k <4 implies a_k + 2<6 $



      The next step I want to put is:



      $frac6 a_k +2 >1$



      However I can only justify this statement if $a_k > -2$ but I can't seem to prove that or find any info in the question to suggest that it.



      Can anyone help me with the proof or my theory?









      share|cite|improve this question












      share|cite|improve this question




      share|cite|improve this question








      edited Aug 2 at 6:37









      ben1806

      33




      33









      asked Aug 2 at 5:48









      user122343

      445




      445




















          2 Answers
          2






          active

          oldest

          votes

















          up vote
          1
          down vote













          Hint:   write it as $;a_n + 1 -4 = dfraca_n-4a_n + 2,$, then (prove and) use that $,a_n+2 gt 0,$.






          share|cite|improve this answer




























            up vote
            0
            down vote













            Assuming you have shown the base case, suppose $0< a_k < 4$ for some $k in mathbbN$. Then we have that $0< a_k + 2 < 6$, which implies
            beginalign*
            frac6a_k + 2 > 1
            endalign*
            (divide both sides of the inequality by $a_k + 2$). Also notice that $frac6a_k + 2 < 3$ since we have assumed $a_k > 0$ and hence $a_k + 2 > 2$.

            beginalign*
            0 < a_k+1 = 5 - frac6a_k + 2 < 4
            endalign*
            as desired.






            share|cite|improve this answer























            • I appreciate your help but can you justify how the implication ( <1) is true? I can't seem to wrap my head around it
              – user122343
              Aug 2 at 6:04










            • If ak was 2 for example , the inequality won't hold
              – user122343
              Aug 2 at 6:09










            • Sorry I made a mistake. It should be 6/(a_k + 2) >1.
              – matt stokes
              Aug 2 at 6:11










            • If ak was -3 wouldn't the statement not hold also . I'm a bit confused on this since I don't know for sure if ak itself ( the terms of the sequence ) is greater than 0 . I assume it is from plugging in values but I feel as though I can't just write that worhout proving for all
              – user122343
              Aug 2 at 6:14










            • Notice a_1 = 1 and the statement holds (this is the base case). Now for the induction part of the proof, if we assume a_n < 4 for arbitrary n, we have just shown that a_n+1<4. This implies that a_n can never be -3.
              – matt stokes
              Aug 2 at 6:19










            Your Answer




            StackExchange.ifUsing("editor", function ()
            return StackExchange.using("mathjaxEditing", function ()
            StackExchange.MarkdownEditor.creationCallbacks.add(function (editor, postfix)
            StackExchange.mathjaxEditing.prepareWmdForMathJax(editor, postfix, [["$", "$"], ["\\(","\\)"]]);
            );
            );
            , "mathjax-editing");

            StackExchange.ready(function()
            var channelOptions =
            tags: "".split(" "),
            id: "69"
            ;
            initTagRenderer("".split(" "), "".split(" "), channelOptions);

            StackExchange.using("externalEditor", function()
            // Have to fire editor after snippets, if snippets enabled
            if (StackExchange.settings.snippets.snippetsEnabled)
            StackExchange.using("snippets", function()
            createEditor();
            );

            else
            createEditor();

            );

            function createEditor()
            StackExchange.prepareEditor(
            heartbeatType: 'answer',
            convertImagesToLinks: true,
            noModals: false,
            showLowRepImageUploadWarning: true,
            reputationToPostImages: 10,
            bindNavPrevention: true,
            postfix: "",
            noCode: true, onDemand: true,
            discardSelector: ".discard-answer"
            ,immediatelyShowMarkdownHelp:true
            );



            );








             

            draft saved


            draft discarded


















            StackExchange.ready(
            function ()
            StackExchange.openid.initPostLogin('.new-post-login', 'https%3a%2f%2fmath.stackexchange.com%2fquestions%2f2869737%2finduction-proof-inequality%23new-answer', 'question_page');

            );

            Post as a guest






























            2 Answers
            2






            active

            oldest

            votes








            2 Answers
            2






            active

            oldest

            votes









            active

            oldest

            votes






            active

            oldest

            votes








            up vote
            1
            down vote













            Hint:   write it as $;a_n + 1 -4 = dfraca_n-4a_n + 2,$, then (prove and) use that $,a_n+2 gt 0,$.






            share|cite|improve this answer

























              up vote
              1
              down vote













              Hint:   write it as $;a_n + 1 -4 = dfraca_n-4a_n + 2,$, then (prove and) use that $,a_n+2 gt 0,$.






              share|cite|improve this answer























                up vote
                1
                down vote










                up vote
                1
                down vote









                Hint:   write it as $;a_n + 1 -4 = dfraca_n-4a_n + 2,$, then (prove and) use that $,a_n+2 gt 0,$.






                share|cite|improve this answer













                Hint:   write it as $;a_n + 1 -4 = dfraca_n-4a_n + 2,$, then (prove and) use that $,a_n+2 gt 0,$.







                share|cite|improve this answer













                share|cite|improve this answer



                share|cite|improve this answer











                answered Aug 2 at 6:46









                dxiv

                53.7k64796




                53.7k64796




















                    up vote
                    0
                    down vote













                    Assuming you have shown the base case, suppose $0< a_k < 4$ for some $k in mathbbN$. Then we have that $0< a_k + 2 < 6$, which implies
                    beginalign*
                    frac6a_k + 2 > 1
                    endalign*
                    (divide both sides of the inequality by $a_k + 2$). Also notice that $frac6a_k + 2 < 3$ since we have assumed $a_k > 0$ and hence $a_k + 2 > 2$.

                    beginalign*
                    0 < a_k+1 = 5 - frac6a_k + 2 < 4
                    endalign*
                    as desired.






                    share|cite|improve this answer























                    • I appreciate your help but can you justify how the implication ( <1) is true? I can't seem to wrap my head around it
                      – user122343
                      Aug 2 at 6:04










                    • If ak was 2 for example , the inequality won't hold
                      – user122343
                      Aug 2 at 6:09










                    • Sorry I made a mistake. It should be 6/(a_k + 2) >1.
                      – matt stokes
                      Aug 2 at 6:11










                    • If ak was -3 wouldn't the statement not hold also . I'm a bit confused on this since I don't know for sure if ak itself ( the terms of the sequence ) is greater than 0 . I assume it is from plugging in values but I feel as though I can't just write that worhout proving for all
                      – user122343
                      Aug 2 at 6:14










                    • Notice a_1 = 1 and the statement holds (this is the base case). Now for the induction part of the proof, if we assume a_n < 4 for arbitrary n, we have just shown that a_n+1<4. This implies that a_n can never be -3.
                      – matt stokes
                      Aug 2 at 6:19














                    up vote
                    0
                    down vote













                    Assuming you have shown the base case, suppose $0< a_k < 4$ for some $k in mathbbN$. Then we have that $0< a_k + 2 < 6$, which implies
                    beginalign*
                    frac6a_k + 2 > 1
                    endalign*
                    (divide both sides of the inequality by $a_k + 2$). Also notice that $frac6a_k + 2 < 3$ since we have assumed $a_k > 0$ and hence $a_k + 2 > 2$.

                    beginalign*
                    0 < a_k+1 = 5 - frac6a_k + 2 < 4
                    endalign*
                    as desired.






                    share|cite|improve this answer























                    • I appreciate your help but can you justify how the implication ( <1) is true? I can't seem to wrap my head around it
                      – user122343
                      Aug 2 at 6:04










                    • If ak was 2 for example , the inequality won't hold
                      – user122343
                      Aug 2 at 6:09










                    • Sorry I made a mistake. It should be 6/(a_k + 2) >1.
                      – matt stokes
                      Aug 2 at 6:11










                    • If ak was -3 wouldn't the statement not hold also . I'm a bit confused on this since I don't know for sure if ak itself ( the terms of the sequence ) is greater than 0 . I assume it is from plugging in values but I feel as though I can't just write that worhout proving for all
                      – user122343
                      Aug 2 at 6:14










                    • Notice a_1 = 1 and the statement holds (this is the base case). Now for the induction part of the proof, if we assume a_n < 4 for arbitrary n, we have just shown that a_n+1<4. This implies that a_n can never be -3.
                      – matt stokes
                      Aug 2 at 6:19












                    up vote
                    0
                    down vote










                    up vote
                    0
                    down vote









                    Assuming you have shown the base case, suppose $0< a_k < 4$ for some $k in mathbbN$. Then we have that $0< a_k + 2 < 6$, which implies
                    beginalign*
                    frac6a_k + 2 > 1
                    endalign*
                    (divide both sides of the inequality by $a_k + 2$). Also notice that $frac6a_k + 2 < 3$ since we have assumed $a_k > 0$ and hence $a_k + 2 > 2$.

                    beginalign*
                    0 < a_k+1 = 5 - frac6a_k + 2 < 4
                    endalign*
                    as desired.






                    share|cite|improve this answer















                    Assuming you have shown the base case, suppose $0< a_k < 4$ for some $k in mathbbN$. Then we have that $0< a_k + 2 < 6$, which implies
                    beginalign*
                    frac6a_k + 2 > 1
                    endalign*
                    (divide both sides of the inequality by $a_k + 2$). Also notice that $frac6a_k + 2 < 3$ since we have assumed $a_k > 0$ and hence $a_k + 2 > 2$.

                    beginalign*
                    0 < a_k+1 = 5 - frac6a_k + 2 < 4
                    endalign*
                    as desired.







                    share|cite|improve this answer















                    share|cite|improve this answer



                    share|cite|improve this answer








                    edited Aug 2 at 6:39


























                    answered Aug 2 at 6:02









                    matt stokes

                    1047




                    1047











                    • I appreciate your help but can you justify how the implication ( <1) is true? I can't seem to wrap my head around it
                      – user122343
                      Aug 2 at 6:04










                    • If ak was 2 for example , the inequality won't hold
                      – user122343
                      Aug 2 at 6:09










                    • Sorry I made a mistake. It should be 6/(a_k + 2) >1.
                      – matt stokes
                      Aug 2 at 6:11










                    • If ak was -3 wouldn't the statement not hold also . I'm a bit confused on this since I don't know for sure if ak itself ( the terms of the sequence ) is greater than 0 . I assume it is from plugging in values but I feel as though I can't just write that worhout proving for all
                      – user122343
                      Aug 2 at 6:14










                    • Notice a_1 = 1 and the statement holds (this is the base case). Now for the induction part of the proof, if we assume a_n < 4 for arbitrary n, we have just shown that a_n+1<4. This implies that a_n can never be -3.
                      – matt stokes
                      Aug 2 at 6:19
















                    • I appreciate your help but can you justify how the implication ( <1) is true? I can't seem to wrap my head around it
                      – user122343
                      Aug 2 at 6:04










                    • If ak was 2 for example , the inequality won't hold
                      – user122343
                      Aug 2 at 6:09










                    • Sorry I made a mistake. It should be 6/(a_k + 2) >1.
                      – matt stokes
                      Aug 2 at 6:11










                    • If ak was -3 wouldn't the statement not hold also . I'm a bit confused on this since I don't know for sure if ak itself ( the terms of the sequence ) is greater than 0 . I assume it is from plugging in values but I feel as though I can't just write that worhout proving for all
                      – user122343
                      Aug 2 at 6:14










                    • Notice a_1 = 1 and the statement holds (this is the base case). Now for the induction part of the proof, if we assume a_n < 4 for arbitrary n, we have just shown that a_n+1<4. This implies that a_n can never be -3.
                      – matt stokes
                      Aug 2 at 6:19















                    I appreciate your help but can you justify how the implication ( <1) is true? I can't seem to wrap my head around it
                    – user122343
                    Aug 2 at 6:04




                    I appreciate your help but can you justify how the implication ( <1) is true? I can't seem to wrap my head around it
                    – user122343
                    Aug 2 at 6:04












                    If ak was 2 for example , the inequality won't hold
                    – user122343
                    Aug 2 at 6:09




                    If ak was 2 for example , the inequality won't hold
                    – user122343
                    Aug 2 at 6:09












                    Sorry I made a mistake. It should be 6/(a_k + 2) >1.
                    – matt stokes
                    Aug 2 at 6:11




                    Sorry I made a mistake. It should be 6/(a_k + 2) >1.
                    – matt stokes
                    Aug 2 at 6:11












                    If ak was -3 wouldn't the statement not hold also . I'm a bit confused on this since I don't know for sure if ak itself ( the terms of the sequence ) is greater than 0 . I assume it is from plugging in values but I feel as though I can't just write that worhout proving for all
                    – user122343
                    Aug 2 at 6:14




                    If ak was -3 wouldn't the statement not hold also . I'm a bit confused on this since I don't know for sure if ak itself ( the terms of the sequence ) is greater than 0 . I assume it is from plugging in values but I feel as though I can't just write that worhout proving for all
                    – user122343
                    Aug 2 at 6:14












                    Notice a_1 = 1 and the statement holds (this is the base case). Now for the induction part of the proof, if we assume a_n < 4 for arbitrary n, we have just shown that a_n+1<4. This implies that a_n can never be -3.
                    – matt stokes
                    Aug 2 at 6:19




                    Notice a_1 = 1 and the statement holds (this is the base case). Now for the induction part of the proof, if we assume a_n < 4 for arbitrary n, we have just shown that a_n+1<4. This implies that a_n can never be -3.
                    – matt stokes
                    Aug 2 at 6:19












                     

                    draft saved


                    draft discarded


























                     


                    draft saved


                    draft discarded














                    StackExchange.ready(
                    function ()
                    StackExchange.openid.initPostLogin('.new-post-login', 'https%3a%2f%2fmath.stackexchange.com%2fquestions%2f2869737%2finduction-proof-inequality%23new-answer', 'question_page');

                    );

                    Post as a guest













































































                    Comments

                    Popular posts from this blog

                    Color the edges and diagonals of a regular polygon

                    Relationship between determinant of matrix and determinant of adjoint?

                    What is the equation of a 3D cone with generalised tilt?